From d39cc2a1c46c592a6a183fa4a345eb5399112477 Mon Sep 17 00:00:00 2001 From: Alexander Kortenko <46195870+HatefulT@users.noreply.github.com> Date: Tue, 16 Jan 2024 21:25:43 +0300 Subject: [PATCH 1/3] Update main.yml --- .github/workflows/main.yml | 6 +++--- 1 file changed, 3 insertions(+), 3 deletions(-) diff --git a/.github/workflows/main.yml b/.github/workflows/main.yml index afc5026..a6bbe95 100644 --- a/.github/workflows/main.yml +++ b/.github/workflows/main.yml @@ -17,13 +17,13 @@ jobs: with: root_file: konspect.tex extra_system_packages: "inkscape" - args: -f -jobname=document -pdf -file-line-error -shell-escape -interaction=nonstopmode -synctex=1 + args: -f -jobname=konspect -pdf -file-line-error -shell-escape -interaction=nonstopmode -synctex=1 - name: Upload PDF file if: always() uses: actions/upload-artifact@v3 with: name: PDF - path: document.pdf + path: konspect.pdf # - name: Create Release # id: create_release # uses: actions/create-release@v1 @@ -58,7 +58,7 @@ jobs: body: build tag_name: pdf files: | - document.pdf + konspect.pdf # - name: Publish HTML to GitHub Pages # uses: peaceiris/actions-gh-pages@v3 # if: always() From 1ec7065596b6a334f1157936498040a2b6c7e58d Mon Sep 17 00:00:00 2001 From: haruspex47 Date: Wed, 17 Jan 2024 13:36:50 +0300 Subject: [PATCH 2/3] ltl-upd --- konspect.tex | 9 ++++++ lection10.tex | 88 +++++++++++++++++++++++++++++---------------------- 2 files changed, 60 insertions(+), 37 deletions(-) diff --git a/konspect.tex b/konspect.tex index 1410f4b..3f34fcf 100644 --- a/konspect.tex +++ b/konspect.tex @@ -91,6 +91,15 @@ } \renewcommand{\qedsymbol}{$\blacksquare$} +\numberwithin{remark}{section} + +\frenchspacing + +\usepackage[labelsep=period]{caption} +\captionsetup{font = small} + + + \begin{document} \pagestyle{plain} diff --git a/lection10.tex b/lection10.tex index e0ef4ce..ade41f3 100644 --- a/lection10.tex +++ b/lection10.tex @@ -17,57 +17,71 @@ % \end{theorem} \begin{ex}[критерий Неймана-Пирсона] - $X_1,\dots,X_n \sim N(a, \sigma)$, $a = a_0$ - известно - $H_0 : \sigma = \sigma_0$ - $H_1 : \sigma = \sigma_1$ + $X_1,\dots,X_n \sim N(a, \sigma)$, $a = a_0$; + $H_0 : \sigma = \sigma_0$, + $H_1 : \sigma = \sigma_1$. \begin{multline*} \dfrac{L(\bar X, \sigma_1)}{L(\bar X, \sigma_0)} - = \prod \dfrac{\dfrac{1}{\sqrt{2\pi} \sigma_1} \exp\left(- \dfrac{(x_k - - a_0)^2}{2\sigma_1^2}\right)}{\dfrac{1}{\sqrt{2\pi} \sigma_0} \exp\left(- \dfrac{(x_k - a_0)^2}{2\sigma_0^2}\right)} - = \left(\dfrac{\sigma_0}{\sigma_1}\right)^n \exp\left(-\dfrac{1}{2\sigma_1^2} \sum (X_k-a_0)^2 + \dfrac{1}{2\sigma_0^2} \sum (X_k-a_0)^2 \right) = \\ - = \left(\dfrac{\sigma_0}{\sigma_1}\right)^n \exp\left( -\dfrac{1}{2} \sum \dfrac{\sigma_0^2 (X_k-a_0)^2 + \sigma_1^2 (X_k - a_0)^2}{\sigma_1^2 \sigma_0^2} \right) \geqslant C + = \prod \dfrac{\dfrac{1}{\sqrt{2\pi} \sigma_1} \exp\left(- \frac{(X_k - + a_0)^2}{2\sigma_1^2}\right)}{\frac{1}{\sqrt{2\pi} \sigma_0} \exp\left(- \frac{(X_k - a_0)^2}{2\sigma_0^2}\right)} + = \left(\dfrac{\sigma_0}{\sigma_1}\right)^n \exp\left(-\frac{1}{2\sigma_1^2} + \sum (X_k-a_0)^2 + \frac{1}{2\sigma_0^2} \sum (X_k-a_0)^2 \right) = \\ + = \left(\dfrac{\sigma_0}{\sigma_1}\right)^n \exp\left( -\frac{1}{2} \sum + \frac{\sigma_0^2 (X_k-a_0)^2 + \sigma_1^2 (X_k - a_0)^2}{\sigma_1^2 + \sigma_0^2} \right) \geqslant C. \end{multline*} - +То есть \begin{equation*} - \left(\dfrac{\sigma_0}{\sigma_1}\right)^n \exp\left(-\dfrac{1}{2} \dfrac{\sigma_0^2 - \sigma_1^2}{\sigma_1^2 \sigma_0^2} \sum (X_k-a_0)^2\right) \geqslant C + \left(\dfrac{\sigma_0}{\sigma_1}\right)^n \exp\left(-\dfrac{1}{2} + \dfrac{\sigma_0^2 + \sigma_1^2}{\sigma_1^2 \sigma_0^2} \sum + (X_k-a_0)^2\right) \geqslant C. \end{equation*} - Если знать, что из $ \sigma_0 $, $ \sigma_1 $ больше, то можно уже построить критерий вида: - $$ S = \left\{ \sum(X_k-a_0)^2 \gtrless C_1 \right\} $$ + Если знать, что из $ \sigma_0 $, $ \sigma_1 $ больше, то можно уже построить + критерий вида + $$ S = \left\{ \sum(X_k-a_0)^2 \gtrless C_1 \right\}. $$ \end{ex} \section{Лекция 10 - 2023-11-08 - Критерий Вальда} -\subsection{Графическая иллюстрация критерия отношения правдоподобия} +% \subsection{Графическая иллюстрация критерия отношения правдоподобия} \begin{figure}[h!] \centering - \includegraphics[width=0.8\textwidth]{Figures/10-plot1.png} - \caption{Иллюстрация критерия Неймана-Пирсона} - \label{fig:10-plot1} + \begin{minipage}{0.45\textwidth} + \centering + \includegraphics[width=0.8\textwidth]{Figures/10-plot1.png} + \caption{Иллюстрация критерия Неймана -- Пирсона} + \label{fig:10-plot1} + \end{minipage}\hfill + \begin{minipage}{0.45\textwidth} + \centering + \includegraphics[width=0.8\textwidth]{Figures/10-plot2.png} + \caption{Иллюстрация критерия Вальда. Снизу область принятия $H_0$; + сверху область принятия $H_1$; + посередине область продолжения наблюдений.} + \label{fig:10-plot2} + \end{minipage} \end{figure} -Таким образом, вывод по верности гипотез может зависеть от объема выборки n. - -\subsection{Графическая иллюстрация критерия Вальда} - -\begin{figure}[h!] - \centering - \includegraphics[width=0.8\textwidth]{Figures/10-plot2.png} - \caption{Иллюстрация критерия Вальда} - \label{fig:10-plot2} -\end{figure} - -На рисунке \ref{fig:10-plot2}: -cнизу область принятия $H_0$; -сверху область принятия $H_1$; -посередине область продолжения наблюдений. -Как только после очередного наблюдения статистика перешла вниз или вверх, наблюдения прекращаются и принимается нужная гипотеза. - -Статистика критерия $(\nu, X_1, X_2, \dots, X_n)$, -$\nu = min \{ n : Z_n \notin (B, A) \}$, -$z_n = \dfrac{L(X_1, \dots, X_n, \theta_1)}{L(X_1, \dots, X_n, \theta_0)}$ - -Критерий: Пусть 0 Date: Tue, 23 Jan 2024 21:31:31 +0300 Subject: [PATCH 3/3] ltl-upd --- lection10.tex | 180 +++++++++++++++++++++++++++---------------------- lection11.tex | 182 ++++++++++++++++++++++++++------------------------ lection12.tex | 104 +++++++++++++++++------------ lection2.tex | 19 +++--- 4 files changed, 265 insertions(+), 220 deletions(-) diff --git a/lection10.tex b/lection10.tex index b4fa624..5a9f123 100644 --- a/lection10.tex +++ b/lection10.tex @@ -27,13 +27,13 @@ = \left(\dfrac{\sigma_0}{\sigma_1}\right)^n \exp\left(-\frac{1}{2\sigma_1^2} \sum (X_k-a_0)^2 + \frac{1}{2\sigma_0^2} \sum (X_k-a_0)^2 \right) = \\ = \left(\dfrac{\sigma_0}{\sigma_1}\right)^n \exp\left( -\frac{1}{2} \sum - \frac{\sigma_0^2 (X_k-a_0)^2 + \sigma_1^2 (X_k - a_0)^2}{\sigma_1^2 + \frac{\sigma_0^2 (X_k-a_0)^2 - \sigma_1^2 (X_k - a_0)^2}{\sigma_1^2 \sigma_0^2} \right) \geqslant C. \end{multline*} То есть \begin{equation*} \left(\dfrac{\sigma_0}{\sigma_1}\right)^n \exp\left(-\dfrac{1}{2} - \dfrac{\sigma_0^2 + \sigma_1^2}{\sigma_1^2 \sigma_0^2} \sum + \dfrac{\sigma_0^2 - \sigma_1^2}{\sigma_1^2 \sigma_0^2} \sum (X_k-a_0)^2\right) \geqslant C. \end{equation*} @@ -82,119 +82,139 @@ \section{Лекция 10 - 2023-11-08 - Критерий Вальда} $\nu = \min \{ n : Z_n \notin (B, A) \}$, $z_n = \frac{L(X_1, \dots, X_n, \theta_1)}{L(X_1, \dots, X_n, \theta_0)}$. \textsc{Критерий}. Пусть $0 rk)$ означает, что $(\ln B < Y_1 + \dots + Y_j < \ln A, j \leqslant rk)$, а следовательно и $(\ln B < \eta_1 + \eta_2 + \dots + \eta_j < \ln A, j \leqslant k)$. + Тогда событие $(\nu > rk)$ означает, что $(\ln B < Y_1 + \dots + Y_j < \ln A,\ + j \leqslant rk)$, а следовательно\footnote{Обратное вообще говоря верно только при $ r=1 $. + Действительно, в ином случае, например, могут нарушаться неравенства $ \ln B + < Y_1 < \ln A$.}, и $(\ln B < \eta_1 + \eta_2 + \dots + \eta_j + < \ln A,\ j \leqslant k)$. +Иными словами, \begin{multline*} P_{\theta_i} (\nu > kr) - = P_{\theta_i} (\ln B < Y_1 + \dots + Y_j < \ln A, j \leqslant kr) \leqslant \\ - \leqslant P_{\theta_i} (\ln B < \eta_1 + \eta_2 + \dots + \eta_i < \ln A, j \leqslant k) \leqslant \\ - \leqslant P_{\theta_i} (|\eta_j| < \ln A - \ln B, j \leqslant k) + = P_{\theta_i} (\ln B < Y_1 + \dots + Y_j < \ln A,\ j \leqslant kr) \leqslant \\ + \leqslant P_{\theta_i} (\ln B < \eta_1 + \eta_2 + \dots + \eta_i < \ln A,\ j + \leqslant k) \leqslant P_{\theta_i} (|\eta_j| < \ln A - \ln B,\ j \leqslant k) =\\ = P_{\theta_i} \left(\bigcap (|\eta_j| < \ln A - \ln B)\right) = + (P_{\theta_i} (|\eta_i| < \ln (A/B)))^k, \end{multline*} - т.к. $\eta_i$ - назависимы и одинаково распределены \\ - \[ - = (P_{\theta_i} (|\eta_i| < \ln (A/B)))^k - \] + поскольку $\eta_i$ назависимы и одинаково распределены. - Выбираем r так, чтобы дисперсия $\eta_1$ была достаточно большой: + Выбираем $ r $ так, чтобы дисперсия $\eta_1$ была достаточно большой. Для + каждого $ i $ \[ - \forall i : M_{\theta_i} \eta_1^2 \geqslant D_{\theta_i} \eta_1 - = r \cdot D_{\theta_i} Y_j > C^2 = (\ln (A/B))^2, + M_{\theta_i} \eta_1^2 \geqslant D_{\theta_i} \eta_1 + = r \cdot D_{\theta_i} Y_j > C^2 = (\ln (A/B))^2. \] Тогда \[ - P(\theta_i) = P_{\theta_i} (|\eta_1| < C) < 1 + P(\theta_i) = P_{\theta_i} (|\eta_1| < C) < 1, \] + иначе $P_{\theta_i} (|\eta_i| < C) = 1 \Rightarrow M \eta_i^2 < C^2$. - иначе $P_{\theta_i} (|\eta_i| < C) = 1 \Rightarrow M \eta_i^2 < C^2$ - - В силу независимости $\eta_j$: + В силу независимости $\eta_j$ имеем \[ - P_{\theta_i} (\nu > rk) \leqslant (P(\theta_i))^k = (P(\theta_i)^{1/r})^{rk} + P_{\theta_i} (\nu > rk) \leqslant (P(\theta_i))^k = + (P(\theta_i)^{1/r})^{rk}. \] - - Тогда, очевидно: + Тогда, очевидно, \[ - P_{\theta_i} (\nu > rk) \to 0, k \to \infty + P_{\theta_i} (\nu > rk) \to 0, k \to \infty. \] - Матож тоже конечно: + Наконец, математическое ожидание аналогично \[ M_{\theta_i} \nu = \sum_{n=1}^{\infty} n P_{\theta_i} (\nu = n) = \sum_{n=1}^{\infty} P_{\theta_i} (\nu \geqslant n) \leqslant \sum_{k=1}^{\infty} ( P(\theta_i)^{1/r} )^{kr} - < \infty + < \infty. \] - \end{proof} -Вторая особенность такого критерия состоит в том, что он чувствителен к порядку учета выборки. +\begin{remark*} +Второй особенностью этого критерия является тот факт, что он чувствителен к порядку +учета выборки. +\end{remark*} \begin{theorem}[Вальда] - Если заданы A и B, то ошибки $\alpha$ и $\beta$ удовлетворяют неравенствам: + Если заданы A и B, то ошибки $\alpha$ и $\beta$ удовлетворяют неравенствам \begin{equation*} \begin{cases} - \dfrac{1 - \beta}{\alpha} \geqslant A \\[1em] - \dfrac{\beta}{1 - \alpha} \leqslant B + \dfrac{1 - \beta}{\alpha} \geqslant A, \\[1em] + \dfrac{\beta}{1 - \alpha} \leqslant B. \end{cases} \end{equation*} - (максимально широкий коридор) - - Эти неравенства называют тождестами Вальда. + (Максимально широкий коридор.) + Эти неравенства называют \emph{тождестами Вальда}. % TODO здесь еще рисуночек (в лекциях на странице 12) \end{theorem} -Применение: если $\alpha$ и $\beta$ заданы, то берут $A = \dfrac{1-\beta}{\alpha}, B = \dfrac{\beta}{1 - \alpha}$ +\textsc{Применение.} Если $\alpha$ и $\beta$ заданы, то берут $A = +\frac{1-\beta}{\alpha}, B = \frac{\beta}{1 - \alpha}$. % TODO расписать в применении, почему берем именно крайние точки из неравенств -Почему так? - иначе сумма ошибок будет больше - -\begin{proof} - $\varkappa_{0n} = \left( \vec{X}_n : B < Z_k < A, k = \overline{1, n-1}, z_n \leqslant B \right)$ - множество тех, которые ведут к принятию гипотезы $H_0$ +Почему так? Иначе сумма ошибок будет больше (см. далее). - $\varkappa_{1n} = \left( \vec{X}_n : B < Z_k < A, k = \overline{1, n-1}, z_n \geqslant A \right)$ - множество тех, которые ведут к принятию гипотезы $H_1$ +\begin{proof} + Положим + \begin{align*} + \varkappa_{0n} &= \left( \vec{X}_n\colon B < Z_k < A,\ k = \overline{1, + n-1},\ Z_n + \leqslant B \right),\\ \varkappa_{1n} &= \left( \vec{X}_n\colon B < Z_k < + A,\ k = \overline{1, n-1},\ Z_n \geqslant A \right) + \end{align*} + --- множества тех выборок, которые ведут к соответственно приёму и отклонению гипотезы $ H_0 $. + % $\varkappa_{1n} = \left( \vec{X}_n : B < Z_k < A, k = \overline{1, n-1}, z_n + % \geqslant A \right)$ --- множество тех, которые ведут к принятию гипотезы + % $H_1$. \begin{equation*} 1 = \sum_n P_{\theta_i} (\nu = n) = \sum_n P_{\theta_i} (\varkappa_{0n}) + \sum_n P_{\theta_i} (\varkappa_{1n}) = \begin{cases} - (1 - \alpha) + \alpha, &\theta_0 \\ - \beta + (1 - \beta), &\theta_1 + (1 - \alpha) + \alpha, &\theta = \theta_0, \\ + \beta + (1 - \beta), &\theta = \theta_1. \end{cases} \end{equation*} - +При этом \[ \alpha = \sum_n P_{\theta_0} (\varkappa_{1n}) \leqslant \dfrac{1}{A} \sum_n P_{\theta_1} (\varkappa_{1n}) - = \dfrac{1 - \beta}{A} + = \dfrac{1 - \beta}{A}. \] - Рассмотрим почему среднее неравенство верно. Для дискретного случая: + Рассмотрим, почему среднее неравенство верно. Для дискретного случая: \begin{multline*} P_{\theta_1} (\varkappa_{0n}) = \sum_{\vec{X_n} \in \varkappa_{1n}} P_{\theta_1}(\xi_1 = X_1, \dots, \xi_n = X_n) = \\ @@ -202,69 +222,69 @@ \section{Лекция 10 - 2023-11-08 - Критерий Вальда} \leqslant \dfrac{1}{A} \sum_{\vec{X_n} \in \varkappa_{1n}} \mathcal{L} (X_1, \dots, X_n, \theta_1) = \dfrac{1 - \beta}{A}, \end{multline*} - то есть верно по построению $\varkappa_{1n}$. - + то есть верно по построению $\varkappa_{1n}$. В свою очередь, \[ \beta = \sum_{n} P_{\theta_1} (\varkappa_{0n}) = \sum_{n=1}^\infty \sum_{\vec{X}_n \in \varkappa_{0n}} P_{\theta_1} (\vec{\xi} - = \vec{X}_n) + = \vec{X}_n). \] \end{proof} -\subsection{Среднее число испытаний в критерии Вальда} +\subsection{Среднее число испытаний в критерии Вальда} % TODO здесь во всей лекции надо поменять обозначение Z_n = ln z_n -Критерий Вальда для выборки $X_1, \dots, X_n, \dots$. +Критерий Вальда для выборки $X_1, \dots, X_n, \dots$ Рассматривается статистика \[ - z_n = \dfrac{\mathcal{L} (X_1, \dots, X_n, \theta_1)}{\mathcal{L} (X_1, \dots, X_n, \theta_0)} + z_n = \dfrac{\mathcal{L} (X_1, \dots, X_n, \theta_1)}{\mathcal{L} (X_1, \dots, + X_n, \theta_0)}. \] -если она выходит из коридора $(B, A)$, прекращаем наблюдения и делаем вывод о верности +Если она выходит из коридора $(B, A)$, прекращаем наблюдения и делаем вывод о верности гипотез. -Обозначим за $\nu$ - номер испытания (или размер выборки, что то же самое), +Обозначим за $\nu$ номер испытания (или размер выборки, что то же самое), на котором закончили наблюдения. - \[ - Y_k = \ln \dfrac{P(X_k, \theta_1)}{P(X_k, \theta_0)} + Y_k = \ln \dfrac{P(X_k, \theta_1)}{P(X_k, \theta_0)}. \] -Критерий $z_\nu = \sum \ln \dfrac{P(X_k, \theta_1)}{P(X_k, \theta_0)} = \sum Y_k$ -\[ - \nu = \min \{ k, z_k \notin (B, A)\} -\] -Сам критерий Вальда тогда: +Построим критерий. Напомним, \begin{align*} - z_\nu &\geqslant \ln A \Rightarrow H_1, \\ - z_\nu &\leqslant \ln B \Rightarrow H_0 + \ln Z_\nu &= \sum_{k=1}^\nu \ln \dfrac{P(X_k, \theta_1)}{P(X_k, \theta_0)} = \sum Y_k,\\ + \nu &= \min \{ k, z_k \notin (B, A)\}. +\end{align*} +Сам критерий Вальда тогда принимает вид +\begin{align*} + \ln z_\nu &\geqslant \ln A \Rightarrow H_1, \\ + \ln z_\nu &\leqslant \ln B \Rightarrow H_0. \end{align*} - - Таблица вероятностей ошибок: \begin{center} \begin{tabular}{|c|c|c|} \hline - z_\nu & \approx \ln A & \approx \ln B \\ + $\ln z_\nu$ & $\approx \ln A$ & $\approx \ln B$ \\ \hline - H_0 & \alpha & 1 - \alpha \\ + $H_0$ & $\alpha$ & $1 - \alpha$ \\ \hline - H_1 & 1 - \beta & \beta \\ + $H_1$ & $1 - \beta$ & $\beta$ \\ \hline \end{tabular} \end{center} -Найдем матожидание числа испытаний +Найдем матожидание числа испытаний. \begin{align*} - M_0 \nu &= \dfrac{M_0 z_\nu}{M_0 Y_k} = \dfrac{\alpha \ln A + (1-\alpha) \ln B}{M_0 Y_k} \\ - M_1 \nu &= \dfrac{M_1 z_\nu}{M_1 Y_k} = \dfrac{(1-\beta) \ln A + \beta \ln B}{M_1 Y_k} + M_0 \nu &= \dfrac{M_0 \ln z_\nu}{M_0 Y_k} = \dfrac{\alpha \ln A + (1-\alpha) + \ln B}{M_0 Y_k}, \\ + M_1 \nu &= \dfrac{M_1 \ln z_\nu}{M_1 Y_k} = \dfrac{(1-\beta) \ln A + \beta \ln + B}{M_1 Y_k}, \end{align*} где $M_0 Y_k$, $M_1 Y_k$ вычисляется уже в конкретном примере из законов распределения. \begin{proof} В соответствии с нашими обозначениями: \[ - z_\nu = \sum_{k=1}^\nu Y_k \Rightarrow M_i z_\nu = M_i \nu M_i Y_k, i = 0, 1 + M_i \ln z_\nu = M_i \nu M_i Y_k, \quad i = 0, + 1. \] - \epigraph{не просто, а очень просто}{Т.~В.~Облакова} - \end{proof} + \epigraph{не просто, а очень просто}{Т.\,В.~Облакова} diff --git a/lection11.tex b/lection11.tex index 5e83642..309676f 100644 --- a/lection11.tex +++ b/lection11.tex @@ -49,23 +49,33 @@ \end{theorem} -\begin{proof} - Напомним определение \emph{характеристической функции}. +% \begin{proof} + Напомним определение \emph{характеристической функции} --- \[ f_\xi (t) = M e^{i t \xi},\qquad f_{\bm\xi} = M e^{i (\mathbf t, \bm \xi)} = - M e^{i \sum t_k \xi_k}. + M e^{i \sum t_k \xi_k} \] + --- и обратим внимание на следующее свойство характеристической функции: + \[ + f_{C\bm\eta} = f_{\bm \xi} (\mathbf t) = M e^{i \mathbf t^T \bm \xi} = M e^{i \mathbf t^T C + \bm\eta} = + M e^{i (C^T \mathbf t)^T \bm \eta} = f_{\bm\eta} (C^T \mathbf t). + \] + В случае $ \xi\sim\mathscr N(\bm\mu, \bm\Sigma) + $ имеем $ f_{\bm\xi}(\mathbf t) = \exp \left( i\bm\mu^T\mathbf t - + \frac{1}{2}\mathbf t^T\bm\Sigma\mathbf t \right) $. + В доказательстве будут использоваться следующие случайные величины: \begin{enumerate} \item Статистика $ \bm \nu = (\nu_1, \nu_2, \ldots, \nu_m)^T$, распределённая полиномиально. Действительно, отождествляя попадание величины $ X_i $ в интервал $ E_j $ с вектором $ \mathbf e(X_i) = \mathbf e_j $, получим $ \bm \nu = \sum \mathbf e(X_i) $. Напомним вероятности этого - закона: - \[ - P(\bm\nu = (n_1, n_2, \dots, n_m)) = \frac{n!}{n_1! n_2! \dots n_m!} - p_1^{n_1} p_2^{n_2} \dots p_m^{n_m}, - \] + закона. + \[ + P(\bm\nu = (n_1, n_2, \dots, n_m)) = \frac{n!}{n_1! n_2! \dots n_m!} + p_1^{n_1} p_2^{n_2} \dots p_m^{n_m}, + \] где, разумеется, $\sum n_i = n$, а \[ \sum \frac{n!}{n_1! n_2! \dots n_m!} p_1^{n_1} \dots p_m^{n_m} = (p_1 + p_2 @@ -79,7 +89,7 @@ \] который является функцией от $ \bm \nu $. Важно заметить, что его скалярный квадрат (квадрат длины) и будет равен статистике $ \chi^2_B $. -\item Наконец, случайный вектор $ \bm \xi = C \nu $, где $ C $ --- некоторая +\item Наконец, случайный вектор $ \bm \xi = C \bm\nu $, где $ C $ --- некоторая ортогональная матрица с заданной первой строчкой (одну единичную строку можно задать в любой ортогональной матрице): \[ @@ -89,94 +99,90 @@ \cdots & \cdots & \cdots \end{pmatrix}. \] - - + Учитывая ортогональность преобразования, получим также $ |\bm\xi|^2 = |\bm\eta|^2 = + \chi^2_B $. \end{enumerate} - - - При нулевой гипотезе - \[ - p_i = P_0 (\xi \in E_i) - \] - - Ясно, что это распределено по полиномиальному закону: - \[ - P(\bar \nu = (n_1, n_2, \dots, n_m)) = \frac{n!}{n_1! n_2! \dots n_m!} p_1^{n_1} p_2^{n_2} \dots p_m^{n_m}, \sum n_i = n - \] - - \[ - \sum \frac{n!}{n_1! n_2! \dots n_m!} p_1^{n_1} \dots p_m^{n_m} = 1 - \] - - \begin{multline*} - f_{\bar \nu} (\bar t) = M e^{i \bar t^T \bar \nu} = - \sum e^{i \sum t_k n_k} \frac{n!}{n_1! n_2! \dots n_m!} p_1^{n_1} \dots p_m^{n_m} = \\ - = \sum \frac{n!}{n_1! n_2! \dots n_m!} (p_1 e^{i t_1})^{n_1} (p_2 e^{i t_2})^{n_2} \dots (p_m e^{i t_m})^{n_m} = - (p_1 e^{i t_1} + p_2 e^{i t_2} + \dots + p_m e^{i t_m})^n = \\ - = (1 + p_1 (e^{i t_1} - 1) + p_2 (e^{i t_2} - 1) + \dots + p_m (e^{i t_m} - 1))^n - \end{multline*} - - \[ - \ln f_{\bar \nu} (\bar t) = n \ln (1 + p_1 (e^{i t_1} - 1) + p_2 (e^{i t_2} - 1) + \dots + p_m (e^{i t_m} - 1)) - \] - - Обозначим - \[ - \eta_k = \frac{\nu_k - np_k}{\sqrt{np_k}}, \chi^2_B = \sum_{k=1}^m \eta_k^2 - \] - - Тогда - \[ - f_{\bar \eta} (\bar t) = M e^{i \bar t^T \bar \eta} = - M e^{i \sum t_k \frac{\nu_k - np_k}{\sqrt{np_k}}} = - M e^{i \sum \frac{t_k}{\sqrt{np_k}} \nu_k} \cdot e^{i \sum t_k \sqrt{np_k}} = - f_{\bar \nu} (\frac{t_1}{\sqrt{np_1}}, \dots, \frac{t_m}{\sqrt{np_m}}) e^{i \sum t_k \sqrt{np_k}} - \] - - Тогда - \[ - \ln f_{\bar \eta} (\bar t) = -i \sum t_k \sqrt{np_k} + n \ln (1+p_1 (e^{\frac{i t_1}{\sqrt{np_1}}} - 1)+ \dots + p_m (e^{\frac{i t_m}{\sqrt{np_m}}} - 1) ) - \] - - Используя эквивалентности, +\begin{proof} + % При нулевой гипотезе + % \[ + % p_i = P_0 (\xi \in E_i) + % \] + % Ясно, что это распределено по полиномиальному закону: + % \[ + % P(\bar \nu = (n_1, n_2, \dots, n_m)) = \frac{n!}{n_1! n_2! \dots n_m!} p_1^{n_1} p_2^{n_2} \dots p_m^{n_m}, \sum n_i = n + % \] + % \[ + % \sum \frac{n!}{n_1! n_2! \dots n_m!} p_1^{n_1} \dots p_m^{n_m} = 1 + % \] + Найдём характеристические функции векторов $ \bm\nu $ и $ \bm\eta $. + \begin{align*} + f_{\bm \nu} (\mathbf t) &= M e^{i \mathbf t^T \bm \nu} = + \sum e^{i \sum t_k n_k} \frac{n!}{n_1! n_2! \ldots n_m!} p_1^{n_1} \ldots p_m^{n_m} = \\ + &= \sum \frac{n!}{n_1! n_2! \ldots n_m!} (p_1 e^{i + t_1})^{n_1} (p_2 e^{i t_2})^{n_2} \ldots (p_m e^{i + t_m})^{n_m} =\\ + &=(p_1 e^{i t_1} + p_2 e^{i t_2} + \ldots + p_m e^{i t_m})^n + = (1 + p_1 (e^{i t_1} - 1) + p_2 (e^{i t_2} - 1) + \ldots + p_m (e^{i t_m} - + 1))^n,\\ + \ln f_{\bm \nu} (\mathbf t) &= n \ln (1 + p_1 (e^{i t_1} - 1) + p_2 (e^{i t_2} + - 1) + \ldots + p_m (e^{i t_m} - 1)). + \end{align*} +% Обозначим +% \[ +% \eta_k = \frac{\nu_k - np_k}{\sqrt{np_k}}, \chi^2_B = \sum_{k=1}^m \eta_k^2 +% \] + Для вектора $ \bm\eta $ имеем + \begin{align*} + f_{\bm \eta} (\mathbf t) &= M \exp{i \mathbf t^T \bm\eta} = + M \exp \left(i \sum t_k \frac{\nu_k - np_k}{\sqrt{np_k}}\right) = + M \exp \left(i \sum \frac{t_k}{\sqrt{np_k}} \nu_k\right) \cdot e^{i \sum t_k + \sqrt{np_k}} =\\ + &=f_{\bm \nu} \left(\frac{t_1}{\sqrt{np_1}}, \dots, \frac{t_m}{\sqrt{np_m}}\right) + \exp\left(i + \sum t_k \sqrt{np_k}\right),\\ + \ln f_{\bm \eta} (\mathbf t) &= -i \sum t_k \sqrt{np_k} + n \ln \left(1+p_1 + \left(\exp\left(\frac{i t_1}{\sqrt{np_1}}\right) - 1\right)+ \dots + p_m \left(\exp\left(\frac{i + t_m}{\sqrt{np_m}}\right) - 1\right) \right), + \end{align*} + или, используя эквивалентности для больших $ n $, \begin{multline*} - = -i \sum t_k \sqrt{np_k} + n \left( \sum p_k \left(e^{\frac{i t_k}{\sqrt{np_k}}} - - 1\right) - \frac{1}{2} \left(\sum p_k(e^{\frac{i t_k}{\sqrt{np_k}}} - 1)\right)^2 + + \ln f_{\bm\eta}(\mathbf t) =\\= -i \sum t_k \sqrt{np_k} + n \left( \sum p_k + \left(\exp\left(\frac{i t_k}{\sqrt{np_k}}\right) + - 1\right) - \frac{1}{2} \left(\sum p_k\left(\exp\left(\frac{i + t_k}{\sqrt{np_k}}\right) - 1\right)\right)^2 + o\left(\frac{1}{n}\right) \right) = \\ = -i \sum t_k \sqrt{np_k} + n \left( \sum p_k \left(\frac{i t_k}{\sqrt{np_k}} + \frac{1}{2} \sum \frac{i t_k}{\sqrt{np_k}}\right)^2 - \frac{1}{2} \left(\sum p_k \frac{i t_k}{\sqrt{np_K}}\right)^2 + o\left(\frac{1}{n}\right) \right) = \\ = - \frac{1}{2} \sum t_k^2 + \frac{1}{2} \left( \sum t_k \sqrt{p_k} \right)^2 + o\left(\frac{1}{n}\right) - = - \frac{1}{2} \bar t^T \bar t + \frac{1}{2} ((C \bar t)_1)^2 + o(1), + = - \frac{1}{2} \mathbf t^T \mathbf t + \frac{1}{2} ((C \mathbf t)_1)^2 + o(1), \end{multline*} - где $C$ - ортогональное преобразование $\bm \xi = C \bm\eta$, с первой - строчкой $(\sqrt{p_1}, \dots, \sqrt{p_m})$, - а индексом 1 в $(C \bar t)_1$ обозначено взятие первого элемента вектора. - - Рассмотрим, что происходит с характеристической функцией при ортогональных преобразованиях: - \[ - f_{C\bm\eta} = f_{\bm \xi} (\mathbf t) = M e^{i \mathbf t^T \bm \xi} = M e^{i \mathbf t^T C - \bm\eta} = - M e^{i (C^T \mathbf t)^T \bm \eta} = f_{\bm\eta} (C^T \mathbf t). - \] - - \[ - \ln f_{\bar \xi} (\bar t) = - \frac{1}{2} (C^T \bar t)^T C^T \bar t + \frac{1}{2} ((C C^T \bar t))^2 + o(1) = -\frac{1}{2} \bar t^T \bar t + \frac{1}{2} t_1^2 + o(1) = - \frac{1}{2} \sum_{k=2}^m t_k^2 + o(1) - \] - - \[ - \chi^2_B = \sum \frac{(\nu_k - np_k)^2}{np_k} = \sum \eta_k^2 = \bar \xi^T C^T C \bar \xi = \sum \xi_k^2 - \] - - Рассмотрим как распределен вектор $\bar \xi$: - - Если бы $\bar \xi \sim N(0, \Sigma)$, то: - \[ - (\xi_1, \dots, \xi_m) \sim N(0, \Sigma) \Leftrightarrow f_{\bar \xi} (\bar t) = e^{-\frac{1}{2} \sum \sigma_k^2 t_k^2} - \] - - В нашей ситуации выходит, что $\sigma_1 = 0$, следовательно, $\sum \xi_k^2 \sim \chi^2 (m-1)$. + где индексом в $(C \mathbf t)_1$ обозначено взятие первой компоненты вектора. + % Рассмотрим, что происходит с характеристической функцией при ортогональных преобразованиях: + % \[ + % f_{C\bm\eta} = f_{\bm \xi} (\mathbf t) = M e^{i \mathbf t^T \bm \xi} = M e^{i \mathbf t^T C + % \bm\eta} = + % M e^{i (C^T \mathbf t)^T \bm \eta} = f_{\bm\eta} (C^T \mathbf t). + % \] + \[ + \ln f_{\bm \xi} (\mathbf t) = - \frac{1}{2} (C^T \mathbf t)^T C^T \mathbf t + + \frac{1}{2} ((C C^T \mathbf t))^2 + o(1) = -\frac{1}{2} \mathbf t^T \mathbf t + + \frac{1}{2} t_1^2 + o(1) = - \frac{1}{2} \sum_{k=2}^m t_k^2 + o(1). + \] + % \[ + % \chi^2_B = \sum \frac{(\nu_k - np_k)^2}{np_k} = \sum \eta_k^2 = \bar \xi^T C^T C \bar \xi = \sum \xi_k^2 + % \] + + Видно, что распределение $ \bm\xi $ в пределе нормально и даже стандартно --- с той + оговоркой, что первая компонента $ \sigma_1 = 0 \Rightarrow \xi_1 \equiv 0$ + вырождена. Тогда $ \sum\xi_k^2 = \chi^2_B \sim \chi^2(m-1) $. + % Посмотрим на распределение $ \bm\xi $. + % Если бы $\bar \xi \sim N(0, \Sigma)$, то: + % \[ + % (\xi_1, \dots, \xi_m) \sim N(0, \Sigma) \Leftrightarrow f_{\bar \xi} (\bar t) = e^{-\frac{1}{2} \sum \sigma_k^2 t_k^2} + % \] + % В нашей ситуации выходит, что $\sigma_1 = 0$, следовательно, $\sum \xi_k^2 \sim \chi^2 (m-1)$. \end{proof} \begin{remark*} diff --git a/lection12.tex b/lection12.tex index 71712ec..e4656d0 100644 --- a/lection12.tex +++ b/lection12.tex @@ -4,37 +4,46 @@ \subsection{Критерий Фишера для сложных гипотез} \begin{ex} Случай сложной гипотезы - $X_1, X_2, \dots, X_n \sim F(x, \bar \theta)$, $\bar\theta = (\theta_1, \dots, \theta_r)$. - - Гипотеза $H_0$ - выборка подчиняется закону $F$. + $X_1, X_2, \dots, X_n \sim F(x, \bm \theta)$, $\bm\theta = (\theta_1, \dots, \theta_r)$. + Гипотеза $H_0$: \emph{выборка подчиняется закону $F$}. \end{ex} -\begin{theorem}[Теорема Фишера (критерий Фишера для сложных гипотез)] - Пусть проверятеся гипотеза $H_0$ - выборка из закона $F(x, \bar\theta)$. - - Пусть множество значений СВ $E = E_1 + \dots + E_l$. +\begin{theorem}[Фишера, или критерий Фишера для сложных гипотез] + Пусть проверятеся гипотеза $H_0$: \emph{выборка из закона $F(x, \bar\theta)$} + и + пусть множество значений СВ $E = E_1 + \dots + E_l$. - Пусть $p_l (\bar\theta) = p(\xi \in E_l) = \int_{E_l} dF(x, \bar\theta)$. - $\hat{\bar\theta}$ - ОМП параметра $\bar\theta$. + Пусть $p_l (\bm\theta) = p(\xi \in E_l) = \int_{E_l} dF(x, \bm\theta)$. + $\hat{\bm\theta}$ есть ОМП параметра $\bm\theta$. Тогда если \[ - \exists \dfrac{\partial p_l(\bar\theta)}{\partial \theta_i}, \dfrac{\partial^2 p_l(\bar\theta)}{\partial \theta_i \partial \theta_j}, i, j = \overline{1, r}; k=\overline{1, l}, + \exists \dfrac{\partial p_l(\bar\theta)}{\partial \theta_i},\ + \dfrac{\partial^2 p_l(\bar\theta)}{\partial \theta_i \partial + \theta_j},\quad i,\, j = \overline{1,\ldots, r};\ k=\overline{1,\ldots, l}, \] - А матрица $\left(\dfrac{\partial p}{\partial \bar\theta}\right)$ - имеет ранг r, то + а матрица $\left(\partial p/\partial \bm\theta\right)$ имеет ранг r, то \[ - \chi^2_B = \sum_{k=1}^l \dfrac{(\nu_k - np_k(\hat{\bar\theta}))^2}{np_k(\hat{\bar\theta})} \toD \chi^2 (l-1-r) + \chi^2_B = \sum_{k=1}^l \dfrac{(\nu_k - + np_k(\hat{\bar\theta}))^2}{np_k(\hat{\bar\theta})} \toD \chi^2 (l-1-r). \] \end{theorem} \begin{remark} - условия теоремы не выполнены например для выборки $X_1, \dots, X_n \sim R[a, b]$. - ОМП: $\hat a = X_{(1)}, \hat b = X_{(n)}$. $p_k (a, b) = \int_{E_k} \dfrac{1}{b-a} dx$ - не является дифф. (в точках a и b скачки). + Условия теоремы не выполнены, например, для выборки $X_1, \dots, X_n \sim + \mathscr R[a, b]$. + Там методом максимального правдоподобия находим $\hat a = X_{(1)}, \hat b = + X_{(n)}$. При этом + \[ + p_k (a, b) = \int_{E_k} \frac{dx}{b-a} + \] + не является + дифференцируемой (в точках $a$ и $b$ скачки). \end{remark} При использовании критерия считают $r = 0$. -Критерий: $\chi^2_B \geqslant \chi^2_{1-\alpha} (l-1-r)$, то $H_0$ отклоняют, если $\chi^2_B < \chi^2_{1-\alpha} (l-1-r)$, то $H_0$ принимается. +\textsc{Критерий}. Если $\chi^2_B \geqslant \chi^2_{1-\alpha} (l-1-r)$, то $H_0$ отклоняют, если $\chi^2_B < \chi^2_{1-\alpha} (l-1-r)$, то $H_0$ принимается. \begin{ex} 537 снарядов упало на Лондон. Территорию Лондона разделили на 576 участков по 0.25 км. @@ -42,29 +51,29 @@ \subsection{Критерий Фишера для сложных гипотез} \begin{center} \begin{tabular}{|c|c|c|c|c|c|c|} \hline - снаряды $k_j$ & 0 & 1 & 2 & 3 & 4 & 7 \\ + снаряды $k_j$ & 0 & 1 & 2 & 3 & \multicolumn{1}{c|}{4} & 7 \\ \hline - участки $\nu_j$ & 229 & 211 & 93 & 35 & 7 & 1 \\ + участки $\nu_j$ & 229 & 211 & 93 & 35 & \multicolumn{2}{c|}{$7 + 1$} \\ \hline - n \hat{p_j} & 226.74 & 211.39 & 98.54 & 30.62 & 8.71 \\ + $n \hat{p_j}$ & 226.74 & 211.39 & 98.54 & 30.62 & + \multicolumn{2}{c|}{8.71}\\ \hline \end{tabular} \end{center} - Гипотеза $H_0$ - число снарядов на 1 участок $\sim Pois$. + Гипотеза $H_0$: \emph{число снарядов на 1 участок $\sim \mathbf{Pois}$}. \[ p(\xi = k) = \dfrac{\lambda^k}{k!} e^{-\lambda}. \] - ОМП $\hat \lambda = \bar X = \dfrac{537}{576}$. Считаем $\hat{p_0} = e^{-537 / 576}, \dots$. - Столбцы 4 и 7 пришлось объединить, так как $n\hat{p_4} < 5$. - +ОМП $\hat \lambda = \bar X = 537/576$. Считаем $\hat{p}_0 = e^{-537 / 576}, \dots$. +Столбцы 4 и 7 пришлось объединить, так как $n\hat{p}_4 < 5$. \[ \chi^2_B \approx 1.5; \chi^2_{0.95} (5-1-r) = \chi^2_{0.95} (3) = 7.81, \] - - Критерий имеет вид: $S = \{ \chi^2_B > \chi^2_{0.95}(3) \}$, $\chi^2_B \notin S \Rightarrow H_0$ -- принимается. + Критерий имеет вид $S = \{ \chi^2_B > \chi^2_{0.95}(3) \}$, и $\chi^2_B \notin + S$ --- значит, $H_0$ принимается. \end{ex} \subsection{Проверка гипотезы о независимости признаков} @@ -97,20 +106,21 @@ \subsection{Проверка гипотезы о независимости пр где $\nu_i = \sum_j v_{ij}$, $\nu_j = \sum_i v_{ij}$. \[ - \chi^2_B = \sum_{i=1}^m \sum_{j=1}^k \dfrac{(\nu_{ij} - n \hat p_{ij})^2}{n \hat p_{ij}} + \chi^2_B = \sum_{i=1}^m \sum_{j=1}^k \dfrac{(\nu_{ij} - n \hat p_{ij})^2}{n + \hat p_{ij}}. \] Параметры: $P(A_1), P(A_2), \dots, P(A_m); \sum P(A_i) = 1$, следовательно, независимых параметров здесь $m-1$. $P(B_1), P(B_2), \dots, P(B_k); \dots$, следовательно, независимых параметров $k-1$. Вместе параметров $r = m+k-2$. \[ - \hat p_{i} = P(A_i) = \frac{\nu_i}{n}; \quad \hat p_j = P(B_j) = \frac{\nu_j}{n}. + \hat p_{i} = P(A_i) = \frac{\nu_i}{n}, \quad \hat p_j = P(B_j) = \frac{\nu_j}{n}. \] -При $H_0$: +При $H_0$ \[ - \hat p_{ij} = \hat p_i \cdot \hat p_j = \dfrac{\nu_i \nu_j}{n^2} + \hat p_{ij} = \hat p_i \cdot \hat p_j = \dfrac{\nu_i \nu_j}{n^2}. \] - +Найдём \begin{multline*} \chi^2_B = \sum_{i=1}^m \sum_{j=1}^k \dfrac{(\nu_{ij} - n \hat p_{ij})^2}{n \hat p_{ij}} @@ -150,14 +160,14 @@ \subsection{Проверка гипотезы о независимости пр \begin{solution} \[ \chi^2_B = 100 \left(\dfrac{15^2}{25\cdot23} + \dfrac{10^2}{45\cdot25} + \dots - + \dfrac{25^2}{55\cdot32} - 1\right) = 44.2 + + \dfrac{25^2}{55\cdot32} - 1\right) = 44.2. \] Не забываем про условие применимости \[ - \dfrac{\nu_i \nu_j}{n} \geqslant 4. + \dfrac{\nu_i \nu_j}{n} \geqslant 4, \] - $\chi^2_{0.95} = 13.3$, следовательно, $H_0$ отклоняем. + при этом $\chi^2_{0.95} = 13.3$. Следовательно, $H_0$ отклоняем. \end{solution} \end{ex} @@ -168,31 +178,37 @@ \subsection{Критерий согласия Колмогорова} Выборочная функция распределения: \[ - \hat{F}_n(x) = \dfrac{1}{n} \sum_{k=1}^n I(x-X_k) + \widehat{F}_n(x) = \dfrac{1}{n} \sum_{k=1}^n I(x-X_k) \] - Если $F(x)$ непрерывна, то закон распределения статистики Колмогорова $D_n = \sup_x |\hat{F}_n(x) - F(x)|$ - не зависит от вида $F(x)$. + Если $F(x)$ непрерывна, то закон распределения статистики Колмогорова + \[ + D_n = + \sup_x |\widehat{F}_n(x) - F(x)| + \] + не зависит от вида $F(x)$. \end{theorem} \begin{proof} - Пусть $F(x)$ - строго монотонна. Перейдем от выборки $X_k$ к выборке $Y_k$ по правилу: + Пусть $F(x)$ строго монотонна. Перейдем от выборки $X_k$ к выборке $Y_k$ по + правилу $Y_k = F(X_k)$. Тогда \[ - X_1, \dots X_n \to Y_1, \dots, Y_n; Y_k = F(X_k) + \dfrac{1}{n} \sum_{k=1}^n I(X_k < F^{-1} (y)) = \dfrac{1}{n} \sum_{k=1}^n + I(Y_k < y), \] - - $\dfrac{1}{n} \sum_{k=1}^n I(X_k < F^{-x} (y)) = \dfrac{1}{n} \sum_{k=1}^n I(Y_k < y)$ - +а \begin{multline*} - D_n = \sup_{-\infty < x < \infty} |\hat{F}_n(x) - F(x)| + D_n = \sup_{-\infty < x < \infty} |\widehat{F}_n(x) - F(x)| = \left|\, \begin{aligned} x = F^{-1} (y) \\ (-\infty, \infty) \to [0, 1] - \end{aligned} \,\right|\\ - = \sup_{0 \leqslant y \leqslant 1} |\hat{F}_n (F^{-1} y) - F(F^{-1} (y))| - = \sup_{0\leqslant y \leqslant 1} |\hat{F}_n(y) - y| + \end{aligned} \,\right| = \\ + = \sup_{0 \leqslant y \leqslant 1} |\widehat{F}_n (F^{-1} (y)) - F(F^{-1} (y))| + = \sup_{0\leqslant y \leqslant 1} |\widehat{F}_n(y) - y|. \end{multline*} - Условие строгой монотонности можно опустить, потому что достижение супремума всё равно достигается на участках строгой монотонности. + Условие строгой монотонности можно опустить, потому что достижение супремума + всё равно имеет место на участках строгой монотонности. \end{proof} О вычислении $D_n$: diff --git a/lection2.tex b/lection2.tex index 51290a8..15bbbd7 100644 --- a/lection2.tex +++ b/lection2.tex @@ -139,20 +139,21 @@ \subsection{Сходимость по вероятности} \end{ex} \begin{theorem} - Если последовательность $\{ \xi_n \}$ монотонно убывает (возрастает) и $\xi_n \toP \xi$, то + Если последовательность $\{ \xi_n \}$ поточечно монотонно убывает (возрастает) и $\xi_n \toP \xi$, то \[ \xi_n \toPN \xi. \] \end{theorem} \begin{proof} -Пусть $ \{\xi_n\} $ монотонно убывает и $ \xi_n \toP 0 $ (в случае возрастания -рассматривается последовательность $ \{\xi_n - \xi\} $). Тогда + Пусть последовательность $ \{\tilde\xi_n\} = \{\xi_n - \xi\} $ монотонно + убывает и $ \tilde\xi_n \toP 0 $ (в случае +возрастания рассматривается последовательность $ \{\xi - \xi_n\} $). Тогда \[ - \sup_{m\geqslant n}|\xi_m - \xi| > \varepsilon = - \sup_{m\geqslant n} |\xi_m| > \varepsilon = \sup_{m\geqslant n} -\xi_m > \varepsilon = \xi_n > \varepsilon. + \{ \omega\colon\sup_{m\geqslant n}|\xi_m - \xi| > \varepsilon\} = + \{\omega\colon \sup_{m\geqslant n} |\tilde\xi_m| > \varepsilon\} = + \{\omega\colon\sup_{m\geqslant n} + \tilde\xi_m > \varepsilon\} = \{\omega\colon\tilde\xi_n > \varepsilon\}. \] - \end{proof} \begin{theorem} @@ -470,8 +471,10 @@ \subsection{Предельные теоремы} \] \end{theorem} +\newpage + \subsection{Резюме} -\begin{figure}[H] +\begin{figure}[h!] \centering \includegraphics[width=0.8\textwidth]{Figures/resume.png} \label{fig:resume}